Đến nội dung

phudinhgioihan nội dung

Có 329 mục bởi phudinhgioihan (Tìm giới hạn từ 10-05-2020)



Sắp theo                Sắp xếp  

#377664 Tìm giá trị lớn nhất của tỉ số giữa 1 số nguyên dương có 4 chữ số và tổng các...

Đã gửi bởi phudinhgioihan on 14-12-2012 - 22:55 trong Bất đẳng thức và cực trị

Tổng quát:
Với mỗi số nguyên dương $n$ có $k$ chữ số $(k>1)$, gọi $s(n)$ là tổng các chữ số của $n$ trong hệ thập phân.
Ứng với mỗi $k>1$, gọi
$$M(k) = \max \left \{ \frac{n}{s(n)}|{10^{k-1} \leq n \leq 10^k-1}\right \};$$
$$m(k) = \min \left \{ \frac{n}{s(n)} | {10^{k-1} \leq n \leq 10^k-1}\right \}$$

Có thể biểu diễn $M(k), m(k)$ theo $k$ hay không?


Giả sử n có dạng $ \overline{a_1a_2...a_k} $ , $ a_1;a_2;..;a_k \in \{0;1;...;9\} ,a_1>0 $

Xét $ P=\dfrac{n}{s(n)}=\dfrac{10^{k-1}a_1+10^{k-2}a_2+...+a_k}{a_1+a_2+...+a_k} \; \; (*)$


$ (*) \Leftrightarrow 0= \sum_{i=1}^k (10^{k-i}-P)a_i \le (10^{k-1}-P) \sum_{i=1}^k a_i$

Do $\sum_{i=1}^k a_i >0 $ nên phải có $ P \le 10^{k-1}$

Cho $a_i=0 \; \forall i>1 $ , khi đó $ P=\dfrac{10^{k-1}a_1}{a_1}=10^{k-1} $

Do đó $ Max P=10^{k-1} $ hay

$$M(k) =10^{k-1} $$


Đặt $ x_i=\frac{a_{i+1}}{a_1} \; , i \in \{1;2...;k-1\} $ và quy ước $x_i =0 ,\;\forall i \le 0 $

$ \Rightarrow 0 \le x_i \le 9 \; , \forall i \in \{1;2...;k-1\} $

Khi đó, $P=\dfrac{10^{k-1}+10^{k-2}x_1+...+x_{k-1}}{1+x_1+x_2+...+10x_{k-2}+x_{k-1}}$

$ \Leftrightarrow P-1= \dfrac{10^{k-1}-1+(10^{k-2}-1)x_1+...+9x_{k-2}}{1+x_1+x_2+...+x_{k-1}} $

Với $k=2 $

$P-1=\dfrac{9}{1+x_1} \ge \dfrac{9}{10} $

$ \Leftrightarrow P \ge \dfrac{19}{10} $, dấu bằng xảy ra khi $n=19 $

Với $ k>2 $

$ P-1 \ge \dfrac{10^{k-1}-1+(10^{k-2}-1)x_1+...+(100-1)x_{k-3}+9x_{k-2}}{10+x_1+x_2+...+x_{k-2}} $

$ \Leftrightarrow \dfrac{P-1}{9} \ge \dfrac{\frac{10^{k-1}-1}{9}+\frac{10^{k-2}-1}{9} x_1+...+11x_{k-3}+x_{k-2}}{10+x_1+x_2+...+x_{k-2}}$

$ \Leftrightarrow \dfrac{P-1}{9}-1 \ge \dfrac{\frac{10^{k-1}-1}{9}-10+\frac{10^{k-2}-10}{9}x_1+...+10x_{k-3}}{10+x_1+x_2+...+x_{k-2}} \ge \dfrac{\frac{10^{k-1}-1}{9}-10+\frac{10^{k-2}-10}{9}x_1+...+110x_{k-4}+10x_{k-3}}{19+x_1+x_2+...+x_{k-3}} \;\; (**) $

Với $k=3$

$\dfrac{P-1}{9}-1 \ge \dfrac{1}{19} $

$\Leftrightarrow P \ge \frac{199}{19} $ , dấu bằng xảy ra khi $n=199$

Nếu ai có Maple thì dễ dàng thấy được trường hợp $k=2;3$ thì quy luật $n_0$ thỏa $m(k)$ có dạng giống nhau và tính chất đã thay đổi kể từ khi $k \ge 4 $ và thuật toán sau sẽ lý giải cho điều khác biệt đó :ukliam2:


Với $k>3 $ , đặt $ k=3+m \; , m \ge 1$ ( ngồi 2 ngày để tạo ra chỗ này nên cứ xem tiếp đã rồi hãy thắc mắc :icon10: )

$(**) \Leftrightarrow \dfrac{P-1}{9}-1 \ge \dfrac{\frac{10^{m+2}-1}{9}-10+\sum_{i=1}^{m} \frac{10^{m+2-i}-10}{9} x_i}{10+9+\sum_{i=1}^{m}x_i} \;\;\; (***)$

Xét khi m rất lớn ( mục đích là để khi thực hiện p bước thuật toán $\Delta $ dưới đây ta vẫn thu được bất đẳng thức đúng ( vì p khá nhỏ so với m nên hệ số tự do trên tử thức dương)! )

Hệ số của $x_m$ trên tử thức là $10$, do đó

$ \dfrac{P-1}{9}-1-10 \ge \dfrac{\frac{10^{m+2}-1}{9}-10-10(10+9)+\sum_{i=1}^{m-1} \frac{10^{m+2-i}-100}{9}x_i}{10+9+\sum_{i=1}^mx_i} $

$\ge \dfrac{\frac{10^{m+2}-1}{9}-10-10(10+9)+\sum_{i=1}^{m-1} \frac{10^{m+2-i}-10^2}{9} x_i}{10+2.9+\sum_{i=1}^{m-1}x_i}$

do $\dfrac{10^{m+2}-1}{9}-10-10(10+9) >0 $ vì m rất lớn và $ x_m \le 9$.


Hệ số của $x_{m-1} $ trên tử thức là $10^2 $, do đó

$ \dfrac{P-1}{9}-1-10-10^2 \ge \dfrac{\frac{10^{m+2}-1}{9}-10-10(10+9)-10^2(10+2.9) +\sum_{i=1}^{m-2} \frac{10^{m+2-i}-10^3}{9} x_i}{10+2.9+\sum_{i=1}^{m-1}x_i} $

$\ge \dfrac{\frac{10^{m+2}-1}{9}-10-10(10+9)-10^2(10+2.9)+\sum_{i=1}^{m-2} \frac{10^{m+2-i}-10^3}{9}x_i}{10+3.9+\sum_{i=1}^{m-2}x_i}$

do $\frac{10^{m+2}-1}{9}-10-10(10+9)-10^2(10+2.9) >0 $ vì m rất lớn và $x_{m-1} \le 9$ .


Đến đây ta đã dần thấy quy luật sau 2 bước của thuật toán sau:

Gọi $\Delta_p$ là bước thứ p của thuật toán $\Delta$ trừ hai vế của bất đẳng thức $(***)$ cho $10^p$ và cho $x_{m-p}=9 $

Ta chứng minh sau p bước ($\frac{10^{m+2}-1}{9} \ge \sum_{i=0}^p 10^i(10+9i)$) thực hiện $\Delta$ , ta thu được bất đẳng thức đúng là:

$\dfrac{P-1}{9}-\sum_{i=0}^p 10^i \ge \dfrac{\frac{10^{m+2}-1}{9}-\sum_{i=0}^p 10^i(10+9i) +\sum_{i=1}^{m-p}\frac{10^{m+2-i}-10^{p+1}}{9}x_i}{10+9(p+1)+\sum_{i=1}^{m-p}x_i} $

Với $p=1;p=2 $ ta đã chứng minh ở trên. Giả sử thu được bất đẳng thức đúng sau khi thực hiện $p$ bước (p thỏa $\frac{10^{m+2}-1}{9} \ge \sum_{i=0}^{p+1} 10^i(10+9i)$), thực hiện trừ 2 vế của bdt cho $10^{p+1} $ ta được bdt :

$\dfrac{P-1}{9}-\sum_{i=0}^{p+1} 10^i \ge \dfrac{\frac{10^{m+2}-1}{9}-\sum_{i=0}^p 10^i(10+9i)-10^{p+1}(10+9(p+1)) +\sum_{i=1}^{m-p-1}( \frac{10^{m+2-i}-10^{p+1}}{9}-10^{p+1}) x_i}{10+9(p+1)+\sum_{i=1}^{m-p}x_i} $

$ \ge \dfrac{\frac{10^{m+2}-1}{9}-\sum_{i=0}^{p+1} 10^i(10+9i) +\sum_{i=1}^{m-p-1}\frac{10^{m+2-i}-10^{p+2}}{9}x_i}{10+9(p+1)+\sum_{i=1}^{m-p}x_i} $

$ \ge \dfrac{\frac{10^{m+2}-1}{9}-\sum_{i=0}^{p+1} 10^i(10+9i) +\sum_{i=1}^{m-p-1}\frac{10^{m+2-i}-10^{p+2}}{9}x_i}{10+9(p+2)+\sum_{i=1}^{m-p-1}x_i} $

do $\frac{10^{m+2}-1}{9}-\sum_{i=0}^{p+1} 10^i(10+9i) >0 $ và $ x_{m-p} \le 9 $

Ta vừa thu được một bất đẳng thức đúng và đây chính là bất đẳng thức thu được với bước $\Delta_{p+1} $

Vậy sau p bước thuật toán $\Delta$ với điều kiện $ \frac{10^{m+2}-1}{9} \ge \sum_{i=0}^p 10^i(10+9i) $ thì ta thu được bất đẳng thức đúng là


$$\dfrac{P-1}{9}-\sum_{i=0}^p 10^i \ge \dfrac{\frac{10^{m+2}-1}{9}-\sum_{i=0}^p 10^i(10+9i) +\sum_{i=1}^{m-p}\frac{10^{m+2-i}-10^{p+1}}{9}x_i}{10+9(p+1)+\sum_{i=1}^{m-p}x_i} \;\;\;(****)$$


Ta có: $\sum_{i=0}^p 10^i(10+9i) =\sum_{i=0}^p 10^i (1+9(i+1) )= \sum_{i=0}^p 10^i +9 \sum_{i=0}^p (i+1) 10^i$

Dễ thu gọn được :

$\sum_{i=0}^p 10^{i}=\dfrac{10^{p+1}-1}{9} $

$9 \sum_{i=0}^p (i+1) 10^i = \dfrac{(p+1)10^{p+2}-(p+2)10^{p+1}+1}{9} $ ( Xét đạo hàm hàm $\sum_{i=0}^p x^{i+1} $ )

Vậy : $\sum_{i=0}^p 10^i(10+9i)=(p+1)10^{p+1} $

Do đó


$$(****) \Leftrightarrow \dfrac{P-1}{9}-\dfrac{10^{p+1}-1}{9} \ge \dfrac{\frac{10^{m+2}-1}{9}-(p+1)10^{p+1} +\sum_{i=1}^{m-p}\frac{10^{m+2-i}-10^{p+1}}{9}x_i}{10+9(p+1)+\sum_{i=1}^{m-p}x_i}$$


Để tìm điểm dừng trong thuật toán, ta phải giải phương trình ẩn p

$\frac{10^{m+2}-1}{9}-(p+1)10^{p+1} \ge 0 $


Đặt $ D=\{ x \in \mathbb{N}, \dfrac{10^{m+2}-1}{9} \ge (x+1) 10^{x+1} \} $

Dễ thấy $ D \neq \varnothing $ vì $ 0 \in D $ $ \Rightarrow \exists p_0=\max D$


Đặt $ A_j=[\dfrac{10^{j+1}-1}{9}+j; \dfrac{10^{j+2}-1}{9}+j] \;\;, j \in \mathbb{N}$ , $ B_j=(\dfrac{10^{j+1}-1}{9}+j; \dfrac{10^{j+2}-1}{9}+j) \;\;, j \in \mathbb{N} $

Dễ thấy rằng $ \cup_{j \in \mathbb{N}} A_j=\mathbb{N}^* $ , do đó $ \exists j \in \mathbb{N}, m \in A_j $

$\forall j_1 \neq j_2 \Rightarrow B_{j_1} \cap B_{j_2}=\varnothing $

do đó, với $m \in \mathbb{N}^* $ cố định cho trước, tồn tại duy nhất $j \in \mathbb{N} $ sao cho $\dfrac{10^{j+1}-1}{9}+j \le m \le \dfrac{10^{j+2}-1}{9}+j $

Với j thỏa điều kiện trên, ta có tính chất ( chưa CM ) : $ p_0=m-j-1 $


Cách khác, sử dụng hàm Lambert , $x=W(x) e^{W(x)} $

Dễ dàng giải được: $ p_0=\left \lfloor \dfrac{\mathbf{W}(\frac{10^{m+2}-1}{9}\ln 10)}{\ln 10} \right \rfloor -1$

Vậy , sau $p_0$ bước thuật toán $\Delta $, ta được bất đẳng thức

$$\dfrac{P-1}{9}-\dfrac{10^{p_0+1}-1}{9} \ge \dfrac{\frac{10^{m+2}-1}{9}-(p_0+1)10^{p_0+1} +\sum_{i=1}^{m-p_0}\frac{10^{m+2-i}-10^{p_0+1}}{9}x_i}{10+9(p_0+1)+\sum_{i=1}^{m-p_0}x_i}$$


$\Leftrightarrow \dfrac{P-1}{9}-\dfrac{10^{p_0+2}-1}{9} \ge \dfrac{\frac{10^{m+2}-1}{9}-(p_0+2)10^{p_0+2} +\sum_{i=1}^{m-p_0-1}\frac{10^{m+2-i}-10^{p_0+2}}{9}x_i}{10+9(p_0+1)+\sum_{i=1}^{m-p_0}x_i} $

$\Leftrightarrow P-10^{p_0+2} \ge \dfrac{10^{m+2}-1-9(p_0+2)10^{p_0+2} +\sum_{i=1}^{m-p_0-1} (10^{m+2-i}-10^{p_0+2})x_i}{10+9(p_0+1)+\sum_{i=1}^{m-p_0}x_i} $

Đặt $f(x_1;x_2;...;x_{m-p_0})=\dfrac{10^{m+2}-1-9(p_0+2)10^{p_0+2} +\sum_{i=1}^{m-p_0-1} (10^{m+2-i}-10^{p_0+2})x_i}{10+9(p_0+1)+\sum_{i=1}^{m-p_0}x_i} $


$f_{x_1}^{'}(x_1;x_2;...;x_{m-p_0}) $

$=\dfrac{(10^{m+1}-10^{p_0+2})(10+9(p_0+1)+\sum_{i=2}^{m-p_0}x_i)-10^{m+2}+1+9(p_0+2)10^{p_0+2}-\sum_{i=2}^{m-p_0-1} (10^{m+2-i}-10^{p_0+2})x_i}{(10+9(p_0+1)+\sum_{i=1}^{m-p_0}x_i)^2}$


$ \ge \dfrac{(10^{m+1}-10^{p_0+2})(10+9(p_0+1)+\sum_{i=2}^{m-p_0}x_i)-10^{m+2}+1+9(p_0+2)10^{p_0+2}-(10^{m+1}-10^{p_0+2})\sum_{i=2}^{m-p_0-1} x_i}{(10+9(p_0+1)+\sum_{i=1}^{m-p_0}x_i)^2} $


$\ge 0$

Do đó,

$f(x_1;x_2;...;x_{m-p_0})$

$\ge f(0;x_2;...;x_{m-p_0})=\dfrac{10^{m+2}-1-9(p_0+2)10^{p_0+2} +\sum_{i=2}^{m-p_0-1} (10^{m+2-i}-10^{p_0+2})x_i}{10+9(p_0+1)+\sum_{i=2}^{m-p_0}x_i} $

$f(x_1;x_2;...;x_{m-p_0}) $ và $f(0;x_2;..;x_{m-p_0}$ có cùng dạng như nhau nên sau khi thực hiện $m-p_0-1$ các bước xét sự biến thiên hàm f theo lần lượt các biến $x_i \;\;\;, i \in \{1;2;...;m-p_0-1\} $

ta thu được

$f(x_1;x_2;...;x_{m-p_0}) \ge f(0;0;..;0;x_{m-p_0}) =\dfrac{10^{m+2}-1-9(p_0+2)10^{p_0+2}}{10+9(p_0+1)+x_{m-p_0}}$


$\ge \dfrac{10^{m+2}-1-9(p_0+2)10^{p_0+2}}{10+9(p_0+1)} $

$\Rightarrow P-10^{p_0+2} \ge \dfrac{10^{m+2}-1-9(p_0+2)10^{p_0+2}}{10+9(p_0+1)} $


$$P \ge \dfrac{10^{m+2}+10^{p_0+2}-1}{9p_0+19} $$

$$\Leftrightarrow {\color{DarkBlue} {P \ge \dfrac{10^{k-1}+10^{p_0+2}-1}{9p_0+19}}} $$

Dấu bằng xảy ra khi $ x_1=...=x_{m-p_0}=0 \;\;, x_{m-p_0+1}=...=x_{k-1}=9 $

hay ${\color{DarkBlue} {m(k)=\dfrac{10^{k-1}+10^{p_0+2}-1}{9p_0+19}} }$

đạt được khi $n=\overline{10..09..9}$ có $ k-p_0-3$ số 0 và $p_0+2$ số 9

Có một điều thú vị là công thức $m(k)$ ở trên đúng với mọi k nguyên dương, tức:



$$m(k)=\dfrac{10^{k-1}+10^{\left \lfloor \dfrac{\textbf{W}(\frac{10^{k-1}-1}{9}\ln 10)}{\ln 10} \right \rfloor +1}-1}{9\left \lfloor \dfrac{\textbf{W}(\frac{10^{k-1}-1}{9}\ln 10)}{\ln 10} \right \rfloor +10} \;\;\; ,\forall k \in \mathbb{N}^* $$


Bài giải có vẻ khủng bố nhưng kết quả thì chắc đúng rồi. Đã test với Maple ! :B)

Thế là đi tông 2 ngày :wacko: :wacko: mà không biết có được 10đ không nữa :lol:

Vẫn tiếp tục tìm công thức tường minh cho $p_0$...



#377456 Tìm giá trị lớn nhất của tỉ số giữa 1 số nguyên dương có 4 chữ số và tổng các...

Đã gửi bởi phudinhgioihan on 14-12-2012 - 01:58 trong Bất đẳng thức và cực trị

Em nhận định rất đúng!
Ban đầu tôi đã đưa ra thuật toán tính số các số $0$ ngay sau số $1$ là
$\left\lfloor\log_{10}(9k)\right\rfloor$
Nhưng thử lại thấy cái này không đúng với $k=12$
Do đó tôi mới đặt ra giả thiết rằng $k\ge 4$ thì $m(k)$ xảy ra tại $n=1099...$
Rồi cố gắng tìm cách quy nạp, nhưng quả thực nó có vấn đề...
Vì vậy ... :))


Nếu em không sai sót thì

$m(k)$ xảy ra tại $n=100....999$ có $k-M-2$ số 0

Với $M=\max \{ x \in \mathbb{N}, x.10^x \le \dfrac{10^{k-1}-1}{9} \}$



#377430 Tìm giá trị lớn nhất của tỉ số giữa 1 số nguyên dương có 4 chữ số và tổng các...

Đã gửi bởi phudinhgioihan on 13-12-2012 - 23:02 trong Bất đẳng thức và cực trị

Max thì dễ rồi ( :P) theo lời giải của thầy Thế $M(k)=10^{k-1}$
Bài toán chỉ còn phải tìm min

Với $k=1$ dễ thấy $m(1)=1$
Với $k=2$ cũng vậy $m(2)=\dfrac{19}{10}$
Với $k=3$ thì $m(3)=\dfrac{199}{19}$
Mấy cái trên cũng không khó khăn lắm để kiểm tra :D. Tuy nhiên chớ có vội mừng ...

Với $k=4$ thì $m(4)=\dfrac{1099}{19}$ cơ!
Ta sẽ chứng minh bằng quy nạp rằng

$m(k)=\mathop\min\limits_{\left\{10^{k-1}\le n\le 10^k-1\right\}}\dfrac{n}{s(n)}=\dfrac{10^{k-1}+10^{k-2}-1}{9k-17}\qquad(k\ge 4)$


Bài giải của chú hxthanh đã sai!

Giá trị $m(k)$ ở trên đạt được khi n có dạng $1099....9$

nhưng với thuật toán em đang làm thì chẳng hạn với $k=15 $, $m(15)$ xảy ra khi $n=100999999999999$ như trong minh họa hình bên dưới, dễ dàng thấy rằng

$\dfrac{100999999999999}{109}<\dfrac{109999999999999}{118} $

Ngày mai em sẽ post lời giải 3 trang cho bài này sau.

Hình đã gửi



#376461 Bộ sách Đại số, Hình học, Giải tích của Jean-Marie Monier

Đã gửi bởi phudinhgioihan on 10-12-2012 - 01:07 trong Tài liệu, chuyên đề Toán cao cấp

Trên kho chia sẻ file me zing của mình


Giải tích 2-Jean Marie Monier

Đại số 2-Jean Marie Monier